Difference between revisions of "2014 AIME I Problems/Problem 12"

(Problem 12)
(Solution)
Line 2: Line 2:
 
Let <math>A=\{1,2,3,4\}</math>, and <math>f</math> and <math>g</math> be randomly chosen (not necessarily distinct) functions from <math>A</math> to <math>A</math>. The probability that the range of <math>f</math> and the range of <math>g</math> are disjoint is <math>\tfrac{m}{n}</math>, where <math>m</math> and <math>n</math> are relatively prime positive integers. Find <math>m</math>.
 
Let <math>A=\{1,2,3,4\}</math>, and <math>f</math> and <math>g</math> be randomly chosen (not necessarily distinct) functions from <math>A</math> to <math>A</math>. The probability that the range of <math>f</math> and the range of <math>g</math> are disjoint is <math>\tfrac{m}{n}</math>, where <math>m</math> and <math>n</math> are relatively prime positive integers. Find <math>m</math>.
  
== Solution ==
+
== Solution ? ==
 +
we note there are <math>4^8</math> sets of two functions  f and g from A to A since the input of the four values of each function has four options each
 +
By the pigeonhole principle the combined range  of <math>f</math> and <math>g</math> has at most four elements
 +
this can be done in 3 cases:
 +
*Case 1: fs range contains 3 elements
 +
there are 4 ways to choose the range of f<math>{{4}\choose {3}}</math> then each element in f can be assigned to one of the 3 elements in the range of f so there are <math>4*</math>3^4<math> functions with a range of 3 elements in A
 +
 
 +
note that g can only be the function assigning each element of A to the unchosen element
 +
 
 +
so there </math>4*<math>3^4</math> for that to happen
 +
 
 +
*Case 2: fs range contains 2 elements
 +
 
 +
there are 6 ways to choose the range of f<math>{{4}\choose {2}}</math> then each element in f can be assigned to one of the 2 elements in the range of f so there are <math>4*2^4</math> functions with a range of 3 elements in A
 +
 
 +
now g can have a range of two elements in which each of its 4 values can be assigned one of its elements (<math>2^4</math> ways)
 +
 
 +
or g can have a range of one element there are 2 ways to choose its range <math>{{2}\choose {1}}</math>  and like in case 1 g can happen in exactly 1 way
 +
 
 +
so there <math>(6*2^4)(2^4+2)</math> ways for that to happen
 +
 
 +
*case 3:  fs range contains 1 element
 +
here are 4 ways to choose the range of f<math>{{4}\choose {1}}</math> then each element in A can be assigned to only one value so there are 4 functions with a range of 1 elements in A
 +
 
 +
now g can have a range of 1,2 or 3:
 +
if g's range has 3 elements each value in A can be assigned to 3 other values so there <math>3^4</math> ways for that to occur.
 +
 
 +
if g's range contains 2 elements one can choose the 2 elements in 3 different ways  <math>{{3}\choose {2}}</math> and after choosing each element has 2 options resulting with <math>2^4</math> ways for that to ocuur
 +
 
 +
if g's range contains 1 element one can choose the 1 elements in 3 different ways  <math>{{3}\choose {1}}</math> and after choosing each element has 1 option resulting with 1 way for that to happen.
 +
so there are 4*(3^4+3*2^4+3) ways for that to occur
 +
 
 +
summing the cases we get that the probability for f and g to have disjoint ranges is equal to:
 +
 
 +
<math>\dfrac{4*3^4+(6*2^4)(2^4+2)+4*(3^4+3*2^4+3)}{4^8}=\dfrac{2^2*3^4+2^6*3^3+2^4*11}{2^{16}}=\dfrac{3^4+2^4*3^3+11*2^2}{2^{14}}=\dfrac{587}{2^{14}}</math>
 +
so the final answer is <math>587</math>

Revision as of 15:03, 14 March 2014

Problem 12

Let $A=\{1,2,3,4\}$, and $f$ and $g$ be randomly chosen (not necessarily distinct) functions from $A$ to $A$. The probability that the range of $f$ and the range of $g$ are disjoint is $\tfrac{m}{n}$, where $m$ and $n$ are relatively prime positive integers. Find $m$.

Solution ?

we note there are $4^8$ sets of two functions f and g from A to A since the input of the four values of each function has four options each By the pigeonhole principle the combined range of $f$ and $g$ has at most four elements this can be done in 3 cases:

  • Case 1: fs range contains 3 elements

there are 4 ways to choose the range of f${{4}\choose {3}}$ then each element in f can be assigned to one of the 3 elements in the range of f so there are $4*$3^4$functions with a range of 3 elements in A

note that g can only be the function assigning each element of A to the unchosen element

so there$ (Error compiling LaTeX. Unknown error_msg)4*$3^4$ for that to happen

  • Case 2: fs range contains 2 elements

there are 6 ways to choose the range of f${{4}\choose {2}}$ then each element in f can be assigned to one of the 2 elements in the range of f so there are $4*2^4$ functions with a range of 3 elements in A

now g can have a range of two elements in which each of its 4 values can be assigned one of its elements ($2^4$ ways)

or g can have a range of one element there are 2 ways to choose its range ${{2}\choose {1}}$ and like in case 1 g can happen in exactly 1 way

so there $(6*2^4)(2^4+2)$ ways for that to happen

  • case 3: fs range contains 1 element

here are 4 ways to choose the range of f${{4}\choose {1}}$ then each element in A can be assigned to only one value so there are 4 functions with a range of 1 elements in A

now g can have a range of 1,2 or 3: if g's range has 3 elements each value in A can be assigned to 3 other values so there $3^4$ ways for that to occur.

if g's range contains 2 elements one can choose the 2 elements in 3 different ways ${{3}\choose {2}}$ and after choosing each element has 2 options resulting with $2^4$ ways for that to ocuur

if g's range contains 1 element one can choose the 1 elements in 3 different ways ${{3}\choose {1}}$ and after choosing each element has 1 option resulting with 1 way for that to happen. so there are 4*(3^4+3*2^4+3) ways for that to occur

summing the cases we get that the probability for f and g to have disjoint ranges is equal to:

$\dfrac{4*3^4+(6*2^4)(2^4+2)+4*(3^4+3*2^4+3)}{4^8}=\dfrac{2^2*3^4+2^6*3^3+2^4*11}{2^{16}}=\dfrac{3^4+2^4*3^3+11*2^2}{2^{14}}=\dfrac{587}{2^{14}}$ so the final answer is $587$